Properties of Functions of Bounded Variation

In summary, the conversation discusses a proof for the bounded variation of a real-valued function on an interval. The proof involves using partitions and a refinement of these partitions. The conversation also mentions a sequence of partitions that is increasing and converges to the variation of the function on the given interval.
  • #1
joypav
151
0
Sorry for all the questions. Reviewing for my midterm next week. Fun fun.
If someone could take a look at my proof for (a) and help me out with (b) that'd be awesome!

(a) Let $\Delta$ be a partition of $[a, b]$ that is a refinement of partition $\Delta'$. For a real-value function $f$ on $[a, b]$, show that $V(f, \Delta') \leq V(f, \Delta)$.

Proof:
Consider
$\Delta' = \left\{x_0, x_1, ... , x_n\right\}$
and a refinement of $\Delta'$
$\Delta = \left\{x_0, x_1, ... , x_n, y\right\}$

Any refinement of $\Delta'$ can be created simply by adding points to $\Delta'$. So, it will suffice to consider the case when one point is added.
Assume, $x_j < y < x_{j+1}$ for some $0 \leq j \leq n-1$.
Then,
$\left| f(x_{j+1}) - f(x_j) \right| = \left| f(x_{j+1}) - f(y) + f(y) - f(x_j) \right| \leq \left| f(x_{j+1}) - f(y) \right| + \left| f(y) - f(x_j) \right|$
(By the triangle inequality)
Then,
$V(f, \Delta') = \sum_{k=0}^{n-1}\left| f(x_{k+1}) - f(x_k) \right| = \left| f(x_{j+1}) - f(x_j) \right| + \sum_{k=0, k \ne j}^{n-1}\left| f(x_{k+1}) - f(x_k) \right| \leq \left| f(x_{j+1}) - f(y) \right| + \left| f(y) - f(x_j) \right| + \sum_{k=0, k \ne j}^{n-1}\left| f(x_{k+1}) - f(x_k) \right| = V(f, \Delta) $

Then the proof is complete when adding one single point and we are done. (If we added more we would simply apply the triangle equality multiple times and repeat the same process.)

(b) Assume $f$ is of bounded variation on $[a, b]$. Show that there is a sequence of partitions $(\Delta_n)$ of $[a, b]$ for which the sequence $V(f, \Delta_n)$ is increasing and converges to $V(f, [a, b])$.
 
Physics news on Phys.org
  • #2
Hi joypav,

Your work for part (a) looks good, nice job.

Part (b) is essentially an induction argument. Here are a few hints to help you with it:
  1. For $n\in\mathbb{N}$, by the definition of a function's variation on an interval as a supremum, there are partitions $\Delta_{n}'$ such that $$V(f,[a,b])-\dfrac{1}{n}< V(f,\Delta_{n}')\leq V(f,[a,b]).$$
  2. Refine the $\Delta_{n}'$ to a new sequence $\Delta_{n}$ so that $\Delta_{n}\subseteq \Delta_{n+1}$ for all $n\in\mathbb{N}.$
  3. Use your result from part (a) to conclude that the sequence of values $V(f,\Delta_{n})$ is increasing and satisfies $$\lim_{n} V(f,\Delta_{n})=V(f,[a,b]).$$
Let me know if anything is still unclear.
 
  • #3
GJA said:
Hi joypav,

Your work for part (a) looks good, nice job.

Part (b) is essentially an induction argument. Here are a few hints to help you with it:
  1. For $n\in\mathbb{N}$, by the definition of a function's variation on an interval as a supremum, there are partitions $\Delta_{n}'$ such that $$V(f,[a,b])-\dfrac{1}{n}< V(f,\Delta_{n}')\leq V(f,[a,b]).$$
  2. Refine the $\Delta_{n}'$ to a new sequence $\Delta_{n}$ so that $\Delta_{n}\subseteq \Delta_{n+1}$ for all $n\in\mathbb{N}.$
  3. Use your result from part (a) to conclude that the sequence of values $V(f,\Delta_{n})$ is increasing and satisfies $$\lim_{n} V(f,\Delta_{n})=V(f,[a,b]).$$
Let me know if anything is still unclear.

Let me see if I'm getting it...

$V(f, [a,b]) = sup_\Delta V(f,\Delta)$
(by definition of supremum) $\implies \forall \epsilon > 0, \exists V(f,\Delta_\epsilon)$ such that,
$V(f, [a,b]) - \epsilon < V(f,\Delta_\epsilon) \leq V(f, [a,b])$

To create our desired sequence, we will let $\epsilon = 1/n$.
Then,
$\forall \epsilon = 1/n > 0, \exists V(f,\Delta_n)$ such that,
$V(f, [a,b]) - 1/n < V(f,\Delta_n) \leq V(f, [a,b])$

Consider the sequence of partitions $(\Delta_n)_{n \in \Bbb{N}}$ of the smallest partition $\Delta_n$ that satisfy this inequality.
As $n$ increases, $1/n$ decreases, $\implies \forall n \in \Bbb{N}, V(f,\Delta_n) \leq V(f,\Delta_{n+1})$.
$\implies$ the sequence $(V(f,\Delta_n))_{n \in \Bbb{N}}$ is increasing.

Then,
$\forall \epsilon > 0$, if $n > 1/\epsilon$,
$\implies \left| V(f,\Delta_n) - V(f,[a,b]) \right| < \left| V(f,[a,b]) - 1/n - V(f,[a,b]) \right| = \left| -1/n \right| = 1/n < \epsilon$

$\implies (V(f,\Delta_n))_{n \in \Bbb{N}} \rightarrow V(f,[a,b])$I'm not sure when I need to refine my partitions $\Delta_n$? Let me know what needs fixing!
 
  • #4
Hi joypav,

joypav said:
To create our desired sequence, we will let $\epsilon = 1/n$.
Then,
$\forall \epsilon = 1/n > 0, \exists V(f,\Delta_n)$ such that,
$V(f, [a,b]) - 1/n < V(f,\Delta_n) \leq V(f, [a,b])$

This is correct. In order to match the notation of part (a), I would recommend calling these $\Delta_{n}'$ because they are not the sequence of partitions we ultimately want to use.

joypav said:
Consider the sequence of partitions $(\Delta_n)_{n \in \Bbb{N}}$ of the smallest partition $\Delta_n$ that satisfy this inequality.
As $n$ increases, $1/n$ decreases, $\implies \forall n \in \Bbb{N}, V(f,\Delta_n) \leq V(f,\Delta_{n+1})$.
$\implies$ the sequence $(V(f,\Delta_n))_{n \in \Bbb{N}}$ is increasing.

This is not correct: $1-\dfrac{1}{n}<V(f,\Delta_{n})\leq V(f,[a,b])$ for all $n$ does not imply $V(f,\Delta_{n})$ is an increasing sequence. For example, $3.5<3.9\leq 4$ could correspond to $n=2$ and $3.\bar{6}<3.75\leq 4$ could correspond to $n=3$ (assuming $V(f,[a,b])=4$, say). But $3.75<3.9,$ so we can't say the sequence is necessarily increasing. This is why you need to refine the partitions and rely on part (a).

Edit. Put another way, the choices you're making for the $\Delta_{n}$ at this point are not related to each other in any way; i.e., what you choose at the $n^{\text{th}}$ step has no bearing on what choice you're making at the $(n+1)^{\text{st}}$ step. This is why we want to proceed inductively rather than randomly. That is, we want to use our knowledge of what occurs at the the $n^{\text{th}}$ step to guide our choice at the $(n+1)^{\text{st}}$ step. Doing so will allow us to meet the extra constraint of creating an increasing sequence, rather than one that simply converges to $V(f,[a,b]).$

joypav said:
Then,
$\forall \epsilon > 0$, if $n > 1/\epsilon$,
$\implies \left| V(f,\Delta_n) - V(f,[a,b]) \right| < \left| V(f,[a,b]) - 1/n - V(f,[a,b]) \right| = \left| -1/n \right| = 1/n < \epsilon$

$\implies (V(f,\Delta_n))_{n \in \Bbb{N}} \rightarrow V(f,[a,b])$

This is true because the $V(f,\Delta_{n})$ satisfy $V(f,[a,b])-\dfrac{1}{n}<V(f,\Delta_{n})\leq V(f,[a,b])$ for all $n$. Note that this step does not require the sequence $V(f,\Delta_{n})$ to be increasing.
 
Last edited:

1. What is the definition of a function of bounded variation?

A function of bounded variation is a real-valued function that has a finite total variation over a given interval. This means that the function has a limited amount of "oscillation" or changes in its values over the interval.

2. How is the total variation of a function calculated?

The total variation of a function is calculated by adding up the absolute differences between consecutive values of the function over a given interval. It can also be expressed as the supremum of the sum of these differences over all possible subintervals.

3. What is the significance of functions of bounded variation?

Functions of bounded variation have important applications in mathematical analysis, particularly in the study of integration and differentiation. They also have practical applications in physics, engineering, and economics.

4. Can a function of bounded variation be discontinuous?

Yes, a function of bounded variation can be discontinuous. The only requirement is that the discontinuities must not be too "large" in terms of their effect on the function's values, meaning that the total variation over the interval must still be finite.

5. Are all continuous functions also functions of bounded variation?

No, not all continuous functions are functions of bounded variation. For example, functions with unbounded derivatives or functions with "jumps" (discontinuities that are too large) are not considered functions of bounded variation, even if they are continuous.

Similar threads

Replies
32
Views
1K
Replies
4
Views
372
  • Topology and Analysis
2
Replies
38
Views
3K
  • Topology and Analysis
Replies
4
Views
1K
  • Topology and Analysis
Replies
14
Views
2K
Replies
2
Views
143
Replies
2
Views
394
  • Topology and Analysis
Replies
3
Views
2K
  • Topology and Analysis
Replies
2
Views
2K
  • Topology and Analysis
Replies
2
Views
1K
Back
Top